• Anúncio Global
    Respostas
    Exibições
    Última mensagem

Limite com raíz cúbica

Limite com raíz cúbica

Mensagempor Rosi7 » Sex Ago 07, 2015 21:34

Gente não estou conseguindo fazer a multiplicação de forma correta, a questão é que mesmo fazendo de forma incorreta minha resposta está batendo com a do livro, pois sempre chego em um numero 1/infinito embaixo = 0, porém tem algo errado.. Eu sinto que tem algo, igual uma questão anterior que eu cortava tudo.. PS: Estou resolvendo o livro leithold por conta própria, não sei ao certo quantas vezes tentei fazer esta questão, mas foram vária e o máximo que chego é na resposta final zero. Embora eu não entendo o que faço na multiplicação, apenas estou usando (a^3-b^3) = a^2 + ab + b^2.

PS: Não posso usar derivada, estou em calculo I e só posso usa-lo na 3 unidade.. ou seja. Se alguém puder me ajudar, peço que seja no tradicional.

\lim_{-\infty}\sqrt[3]{{x}^{3} + x} - \sqrt[3]{{x}^{3} + 1}
Rosi7
Usuário Ativo
Usuário Ativo
 
Mensagens: 15
Registrado em: Sáb Mai 02, 2015 18:49
Formação Escolar: GRADUAÇÃO
Área/Curso: Licenciatura em Física
Andamento: cursando

Re: Limite com raíz cúbica

Mensagempor nakagumahissao » Sáb Ago 08, 2015 12:54

\lim_{x \rightarrow \infty}\sqrt[3]{{x}^{3} + x} - \sqrt[3]{{x}^{3} + 1}

Veja bem, não é possível usar derivadas neste caso porque o problema não trata de uma indefinição que possivelmente você verá num futuro próximo. Temos que trabalhar com essas raízes de outra forma.

Vamos usar a seguinte identidade:

a^2 - b^2 = (a-b)(a+b) \Rightarrow a-b = \frac{a^2 - b^2}{a + b}

Sendo que:

a = \sqrt{x^3 + x}

b = \sqrt{x^3 + 1}

Assim:

\lim_{x \rightarrow \infty} \sqrt[3]{{x}^{3} + x} - \sqrt[3]{{x}^{3} + 1} = \lim_{x \rightarrow \infty} \frac{\left(\sqrt[3]{{x}^{3} + x} \right)^{2} - \left(\sqrt[3]{{x}^{3} + 1} \right)^2}{\sqrt[3]{{x}^{3} + x} + \sqrt[3]{{x}^{3} + 1}} =

= \lim_{x \rightarrow \infty} \frac{\left(x\sqrt[3]{1 + \frac{x}{{x}^{3}}} \right)^{2} - \left(x\sqrt[3]{1 + \frac{1}{{x}^{3}}} \right)^2}{x\sqrt[3]{1 + \frac{x}{{x}^{3}}} + x\sqrt[3]{1 + \frac{1}{{x}^{3}}}} = \frac{x^2 - x^2}{x + x} = \frac{0}{2x} = 0

Lembrando que (Explicarei o que ocorreu somente com a primeira raiz cúbica para que entenda a linha acima):

\sqrt[3]{{x}^{3} + x} = \sqrt[3]{\frac{{x}^{3}}{{x}^{3}}({x}^{3} + x)} = \sqrt[3]{{x}^{3} \left(1 + \frac{x}{x^3} \right)}  =

= \sqrt[3]{{x}^{3} } \cdot \sqrt[3]{\left(1 + \frac{x}{x^3} \right)} = x\sqrt[3]{\left(1 + \frac{x}{x^3} \right)}

\blacksquare
Eu faço a diferença. E você?

Do Poema: Quanto os professores "fazem"?
De Taylor Mali
nakagumahissao
Colaborador Voluntário
Colaborador Voluntário
 
Mensagens: 386
Registrado em: Qua Abr 04, 2012 14:07
Localização: Brazil
Formação Escolar: GRADUAÇÃO
Área/Curso: Lic. Matemática
Andamento: cursando

Re: Limite com raíz cúbica

Mensagempor Rosi7 » Seg Ago 10, 2015 13:22

Entendi onde é meu erro. Embaixo eu não repetia, fiz a regra do a² +ab + b² também.. Que confusão a minha!!!

Muitíssimo obrigada!!!!!!!!

Obs: Nakagumahissao, notei que você usou a²-b², posso usar isso? Sendo que tenho raiz cúbica o certo não seria a³-b³?
Rosi7
Usuário Ativo
Usuário Ativo
 
Mensagens: 15
Registrado em: Sáb Mai 02, 2015 18:49
Formação Escolar: GRADUAÇÃO
Área/Curso: Licenciatura em Física
Andamento: cursando


Voltar para Cálculo: Limites, Derivadas e Integrais

 



  • Tópicos relacionados
    Respostas
    Exibições
    Última mensagem

Quem está online

Usuários navegando neste fórum: Nenhum usuário registrado e 48 visitantes

 



Assunto: Exercicios de polinomios
Autor: shaft - Qua Jun 30, 2010 17:30

2x+5=\left(x+m\right)²-\left(x-n \right)²

Então, o exercicio pede para encontrar {m}^{3}-{n}^{3}.

Bom, tentei resolver a questão acima desenvolvendo as duas partes em ( )...Logo dps cheguei em um resultado q nao soube o q fazer mais.
Se vcs puderem ajudar !


Assunto: Exercicios de polinomios
Autor: Douglasm - Qua Jun 30, 2010 17:53

Bom, se desenvolvermos isso, encontramos:

2x+5 = 2x(m+n) + m^2-n^2

Para que os polinômios sejam iguais, seus respectivos coeficientes devem ser iguais (ax = bx ; ax² = bx², etc.):

2(m+n) = 2 \;\therefore\; m+n = 1

m^2-n^2 = 5 \;\therefore\; (m+n)(m-n) = 5 \;\therefore\; (m-n) = 5

Somando a primeira e a segunda equação:

2m = 6 \;\therefore\; m = 3 \;\mbox{consequentemente:}\; n=-2

Finalmente:

m^3 - n^3 = 27 + 8 = 35

Até a próxima.